From f5cd4b106eedf9240af3af1359f6d23a63d51fa1 Mon Sep 17 00:00:00 2001 From: "W. Trevor King" Date: Tue, 7 Jul 2009 21:52:57 -0400 Subject: [PATCH] Added recitation week 3 recitation solutions --- .../Young_and_Freedman_12/problem23.21.tex | 33 ++++++++++ .../Young_and_Freedman_12/problem23.22.tex | 60 +++++++++++++++++ .../Young_and_Freedman_12/problem23.25.tex | 65 +++++++++++++++++++ .../Young_and_Freedman_12/problem23.27.tex | 18 ++++- .../Young_and_Freedman_12/problem23.58.tex | 28 ++++++++ .../Young_and_Freedman_12/problem23.60.tex | 22 +++++++ .../Young_and_Freedman_12/problem23.63.tex | 54 +++++++++++++++ 7 files changed, 279 insertions(+), 1 deletion(-) diff --git a/latex/problems/Young_and_Freedman_12/problem23.21.tex b/latex/problems/Young_and_Freedman_12/problem23.21.tex index ca5cb0d..63b5dd5 100644 --- a/latex/problems/Young_and_Freedman_12/problem23.21.tex +++ b/latex/problems/Young_and_Freedman_12/problem23.21.tex @@ -42,4 +42,37 @@ qO.draw(); qT.draw(); \end{problem*} \begin{solution} +\Part{a} +One of the nice features of electric potential is that it is a scalar, +so it's a lot easier to sum up then the vector electric field. +\begin{equation} + V_A = V_{1A} + V_{2A} + = \frac{kq_1}{r_{1A}} + \frac{kq_2}{r_{2A}} + = 8.99\E{9}\U{Nm$^2$/C$^2$}\cdot\p({ + \frac{2.40\E{-9}\U{C}}{0.050\U{m}}+\frac{-6.5\E{-9}\U{C}}{0.050\U{m}}}) + = \ans{-737\U{J/C}} \;. +\end{equation} + +\Part{b} +\begin{equation} + V_B = V_{1B} + V_{2B} + = \frac{kq_1}{r_{1B}} + \frac{kq_2}{r_{2B}} + = 8.99\E{9}\U{Nm$^2$/C$^2$}\cdot\p({ + \frac{2.40\E{-9}\U{C}}{0.080\U{m}}+\frac{-6.5\E{-9}\U{C}}{0.060\U{m}}}) + = \ans{-704\U{J/C}} \;. +\end{equation} + +\Part{c} +The electric potential energy change of the moving charge is given by +\begin{equation} + \Delta U_{BA} = q\Delta V_{BA} = q(V_A-V_B) + = 2.50\E{-9}\U{C}\cdot (-737 + 704)\U{J/C} + = -82\U{nJ} \;. +\end{equation} +It makes sense that the charge lost electric potential energy, since +it is a positive charge moving to the lower potential point $A$. The +electric potential energy lost went into some other form of energy +(kinetic, heat, work, mechanical), but the electric field was giving +energy to that other form, so it does \emph{positive} work: +$\ans{82\U{nJ}}$. \end{solution} diff --git a/latex/problems/Young_and_Freedman_12/problem23.22.tex b/latex/problems/Young_and_Freedman_12/problem23.22.tex index c9dd3ac..6861f01 100644 --- a/latex/problems/Young_and_Freedman_12/problem23.22.tex +++ b/latex/problems/Young_and_Freedman_12/problem23.22.tex @@ -14,4 +14,64 @@ $x\gg a$? Explain why this result is obtained. \end{problem*} \begin{solution} +\Part{a} +\begin{center} +\begin{asy} +import Mechanics; +import ElectroMag; + +real u = 1cm; +real a = 1; + +Charge qt = pCharge((0,a)*u); +Charge qb = pCharge((0,-a)*u); + +qt.draw(); qb.draw(); +draw_ijhat((0,0)); +\end{asy} +\end{center} + +\Part{b} +This is particularly easy, since both scalar electric potentials are +the same (same $q$, same $r=a$, direction doesn't matter). +\begin{equation} + V_0 = V_{0t} + V_{0b} = 2V_{0t} = \ans{\frac{2kq}{a}} \;. +\end{equation} + +\Part{c} +The $x$-axis is still symmetric (same distance to each charge), so +$V=2V_t$, and we can use the Pythagorean theorem for $r$. +\begin{equation} + V = 2V_t = \frac{2kq}{r} = \frac{1}{4\pi\varepsilon_0}\frac{2q}{r} + = \ans{\frac{1}{4\pi\varepsilon_0}\frac{2q}{\sqrt{a^2+x^2}}} \;. +\end{equation} + +\Part{c} +\begin{center} +\begin{asy} +import graph; +size(5cm, 2cm, IgnoreAspect); + +real a=1; +real V(real x){ + return 1.0/(a**2 + x**2)**0.5; +} + +xaxis("$x$"); +yaxis("$V$"); +draw(graph(V, -2*a, 2*a, n=600), red); +label("$a$", align=S, Scale((a,0))); +label("$-a$", align=S, Scale((-a,0))); +\end{asy} +\end{center} + +\Part{e} +As $x$ gets much bigger than $a$, $a^2+x^2\rightarrow x^2$, so +\begin{equation} + V = \frac{2kq}{r^2} \approx \ans{\frac{k\cdot2q}{x^2}} \;. +\end{equation} +In other words, the electric field is pretty much the same as it would +be if both charges were sitting at the origin. This makes sense since +the distinction that the charge is actually some small distance $a$ +off the origin isn't very important for $x\gg a$. \end{solution} diff --git a/latex/problems/Young_and_Freedman_12/problem23.25.tex b/latex/problems/Young_and_Freedman_12/problem23.25.tex index d98b091..c97757b 100644 --- a/latex/problems/Young_and_Freedman_12/problem23.25.tex +++ b/latex/problems/Young_and_Freedman_12/problem23.25.tex @@ -12,4 +12,69 @@ result is obtained. \end{problem*} \begin{solution} +\begin{center} +\begin{asy} +import Mechanics; +import ElectroMag; + +real u = 1cm; +real a = 1; + +Charge p = pCharge((0,0), L="$q$"); +Charge n = pCharge((a,0)*u, L="$-2q$"); + +draw_ijhat((0,0)); +p.draw(); n.draw(); +\end{asy} +\end{center} + +\Part{b} +\begin{equation} + V = V_+ + V_- = \frac{kq}{|x|} + \frac{-2kq}{|x-a|} + = kq \p({\frac{1}{|x|} - \frac{2}{|x-a|}}) \;. +\end{equation} + +\Part{c} +$kq > 0$ so $V=0$ only when +\begin{align} + && 0 &= \frac{1}{|x|} - \frac{2}{|x-a|} \\ + && 2|x| &= |x-a| \\ + -2x &= a-x & 2x &= a-x & 2x &= x-a \\ + x &= -a & x &= \frac{a}{3} & x &= -a < 0 \\ + x &= \ans{-a} & x &= \ans{\frac{a}{3}} & &\text{contradiction} \\ +\end{align} +Where the split to three bands comes from taking all possible absolute +value conditions ($x<0